Site hosted by Angelfire.com: Build your free website today!

LSAT

Logic Games Answers & Explanations

 

The Action:
You have to sequence trophies on Shelves 1, 2, and 3, from top to bottom.

The Rules:
2) Rule 2 seems to be the most helpful, so let's look at it first. F must be on the shelf immediately above the shelf that L is on. You have two basic options. In Option 1 you place F on Shelf 1 and L on Shelf 2. With Option 2 you put F of Shelf 2 and L on Shelf 3.
1) In Option 1, we can write next to Shelf 2 "no J," and in Option 2, we can write next to Shelf 3 "no J."
3) No shelf can hold all three bowling trophies.
4) K can't be on Shelf 2 — that's for either option.

1. B
G and H are on Shelf 2, so if you remember that three bowling trophies can't be on the same shelf, this tells us that we must work with Option 1. If you put G and H on Shelf 2 in Option 2, you'd be breaking Rule 3 — you'd have all 3 bowling trophies on the same shelf. So you'll have F on Shelf 1, and L, G, and H on Shelf 2. What must be true? Take a look at B, L is on Shelf 2. Yes we just went through the deduction whereby you realize you must use Option 1 in which F is on Shelf 1 and L is on Shelf 2. So B is the correct answer.

2. D
Right away we realize that you can't use Option 2 here because Option 2 already has a tennis trophy on Shelf 3, L, so you will work with Option 1 — F on the first shelf and L on the second shelf. You know that neither J nor K can appear on Shelf 2 in Option 1. J and K are tennis trophies, so if the question specifies that you can't have a tennis trophy on Shelf 3 and you can't have these two trophies on Shelf 2, then the only place for them is on Shelf 1. In other words, K and J must be on the same shelf, so D is correct.

3. C
This question is directing you to Option 2, because you already know that J isn't allowed on Shelf 2 in Option 1. With Option 2 you know that F must appear on Shelf 2, so C is correct.

4. D
In only one option can Shelf 1 remain empty, Option 2. The rest of the question says "Which of the following must be false?" which means "Which of the following arrangements won't work?" First, let's look at the basic situation. We have Option 2 and we have F on 2 and L on 3, and Shelf 1 remains empty. That tells us that we can do something with J and K. We know in Option 2 that J can't go on Shelf 3 and Shelf 1 is empty, so the only other place for it is Shelf 2. We know that K can't be on Shelf 2 and Shelf 1 is emphy, so the only home of K is Shelf 3. So we have Shelf 1 empty, Shelf 2 with F and J, and Shelf 3 with L and K. What to do with G and H? The only thing we can't do is put them on Shelf 2 because that would violate Rule 3. So if we keep them together we have to put them on Shelf 3. If we split them up, we can put G on 2 and H of Shelf 3, or vice versa.

A — can we put H and F on the same shelf? Sure, we've already said we can put one of G and H on Shelf 2 and one on Shelf 3. B — can we put exactly three trophies on Shelf 2? Sure, we just did with A. We put F, J, and H together on Shelf 2 and that left us with L, K, and G together on Shelf 3. C — can we put G and H on the same shelf? Yes, as long as they're on Shelf 3 and not on Shelf 2. D — can we put exactly two trophies on Shelf 3? We have L and K on Shelf 3. To have exactly two trophies on Shelf 3, we would put both G and H somewhere else and we can't put G and H together on Shelf 2 because that would violate Rule 3. So D is our answer here — it's the thing we can't do. E — can we put G and K on the same shelf? Yes, whether G is alone or together with H, it's possible to do this.

5. A
This is hard because the if-clause doesn't narrow it down to one of the two options. L and G can be on the same shelf in both options, which make s your work more complicated. In both options there's just one empty shelf — in Option 1 it's Shelf 3, and in Option 2 it's Shelf 1. Let's see if we can make any ore deductions about both options. In Option 1, if we have to leave Shelf 3 empty, we can figure out what to do with K and J because they can't be on Shelf 2 and Shelf 3 is empty, so Shelf 1 has F, K, and J and Shelf 2 has L and G and the only thing left is H, on either Shelf 1 or Shelf 2. In Option 2 we know that K can't be on Shelf 2, and Shelf 1 has to be empty, so the only place for K is Shelf 3. J can't be on Shelf 3 in Option 2, and Shelf 1 is empty, so J is on Shelf 2. So we end up with F and J on Shelf 2, L, K, and G on Shelf 3, and Shelf 1 empty, and H is a floater.

For the answer to be correct, it must be true in both options — you hit pay dirt right away, because A is correct. It says if H is on Shelf 3, then J is on Shelf 2. The only way to put H on Shelf 3 is Option 2, where Shelf 3 is open. You can put H on Shelf 3, and in Option 2, J is on Shelf 2, so A is correct. B describes K and L as being on the same shelf, but that's true only in Option 2. C says if H is on Shelf 2, J is on Shelf 3, but J is never on Shelf 3. D has F and K on the same shelf; that's tru in Option 1 only and not in Option 2. And E has J on Shelf 2. That's Option 2, but it goes on to say that H is on Shelf 1, and in Option 2, Shelf 1 is empty. So A is correct.

Back to Home


Back to Questions

Other PracticeQs:
Bullet   Logical Reasoning
Bullet   Reading Comprehension